Cardiology Test

Unlock all answers in this set

Unlock answers
question
Junctional escape rhythms are consistently characterized by? A. Absence of P waves B. QRS complexes greater than 0.12 secs. C. inverted P waves before QRS D. a ventricular rate of 40 - 60
answer
D. A ventricular rate of 40 - 60
question
The administration of dopamine or any other vasopressor drug requires: A. online medical control approval B. careful titration and blood pressure monitoring C. an electromechanical infusion pump D. concomitant crystalloid fluid boluses
answer
B. Careful titration and monitoring
question
To increase myocardial contractility and heart rate and to relax the bronchial smooth muscle, you must give a drug that: A. stimulates beta-1 and beta-2 receptors. B. stimulates beta-2 and alpha receptors. C. blocks beta-1 and beta-2 receptors. D. blocks beta receptors and stimulates alpha receptors.
answer
A. Stimulates beta-1 and beta-2 receptors
question
Which layer of the blood vessel is made up of elastic fibers and muscle, and provides for strength and contractility? a. Tunica media b. Tunica intima c. Tunica adventitia d. Arterial lumen
answer
a. Tunica media
question
Infarctions of the inferior myocardial wall are MOST often caused by: a. blockage of the left coronary artery. b. acute spasm of the circumflex artery. c. occlusion of the right coronary artery. d. a blocked left anterior descending artery.
answer
c. occlusion of the right coronary artery.
question
You have applied the defibrillator pads to a pulseless and apneic 60-year-old woman and observe a slow, wide QRS complex rhythm. Your next action should be to: a. attempt cardiac pacing. b. check the carotid pulse. c. assess breathing effort. d. immediately begin CPR.
answer
d. immediately begin CPR.
question
Injury to the inferior wall of the myocardium would present with: a. T wave inversion in leads V1 through V4. b. ST segment elevation in leads II, III, and aVF. c. pathologic Q waves in leads V4 and V5. d. ST segment depression in leads V5, V6, and aVL.
answer
b. ST segment elevation in leads II, III, and aVF.
question
In contrast to coarse ventricular fibrillation, fine ventricular fibrillation indicates that: a. energy reserves of the cardiac cells have been expended. b. the arrhythmia is more likely to respond to defibrillation. c. a perfusing rhythm is not possible following defibrillation. d. the cardiac cells temporarily have adequate energy stores.
answer
a. energy reserves of the cardiac cells have been expended.
question
Q waves are considered abnormal or pathologic if they are: a. greater than 0.02 seconds wide and consistently precede the R wave. b. deeper than one third the total height of the QRS complex in lead II. c. not visible in leads I or II when the QRS gain sensitivity is increased. d. present in a patient who is experiencing chest pressure or discomfort.
answer
b. deeper than one third the total height of the QRS complex in lead II.
question
The appropriate initial dose of morphine sulfate is: a. 0.5 mg/kg b. 1 to 2 mg c. 2 to 3 mg d. 2 to 4 mg
answer
d. 2 to 4 mg
question
Atrial kick is MOST accurately defined as: a. the blood that flows passively into the ventricles. b. pressure on the atrioventricular valves during ventricular contraction. c. an attempt of the atria to contract against closed valves. d. 20% of ventricular filling caused by atrial contraction.
answer
d. 20% of ventricular filling caused by atrial contraction.
question
You have restored spontaneous circulation in a 54-year-old man who was in ventricular fibrillation. During the arrest interval, you delivered 2 shocks, 1 mg of epinephrine, and 300 mg of amiodarone. The patient's blood pressure is 96/60 mm Hg and the cardiac monitor displays a sinus rhythm at a rate of 70 beats/min with frequent premature ventricular complexes. Appropriate post-resuscitation care for this patient includes: a. 0.5 mg of atropine sulfate. b. an infusion of amiodarone. c. a 20 mL/kg crystalloid bolus. d. a low-dose dopamine infusion.
answer
b. an infusion of amiodarone.
question
Which of the following leads provides the BEST view of the anterolateral wall of the left ventricle? a. V2 to V3 b. V4 to V6 c. V4 to V5 d. V5 to V6
answer
b. V4 to V6
question
Thousands of fibrils that are distributed throughout the ventricles, which represent the end of the cardiac condition system, are called the: a. bundle branches. b. internodal pathways. c. purkinje fibers. d. cardiac myocytes.
answer
c. purkinje fibers.
question
Repolarization begins when: a. the sodium and calcium channels close. b. calcium ions slowly enter the cardiac cell. c. potassium ions rapidly escape from the cell. d. the inside of the cell returns to a positive charge.
answer
a. the sodium and calcium channels close.
question
You receive a call to a residence for a 44-year-old man who is \"ill.\" The patient, who receives dialysis treatments three times a week, tells you that he has missed his last two treatments because he was not feeling well. As your partner takes the patient's vital signs, you apply the ECG, which reveals a sinus rhythm with tall T waves. The 12-lead ECG reveals a sinus rhythm with inverted complexes in lead aVR. On the basis of your clinical findings, you should be MOST suspicious that the patient is: a. hypocalcemic. b. hypernatremic. c. hyperkalemic. d. having an acute MI.
answer
c. hyperkalemic.
question
A delta wave is identified on a cardiac rhythm strip as a/an: a. apparent P wave that occurs at the end of the QRS complex. b. acute widening of the QRS complex immediately after the R wave. c. rapid up slope to the R wave immediately after the end of the P wave. d. delay between the end of the P wave and the beginning of the R wave.
answer
c. rapid up slope to the R wave immediately after the end of the P wave.
question
Blood which has collected carbon dioxide and wastes from the coronary circulation drains into the: a. coronary sinus. b. coronary veins. c. coronary arteries. d. coronary sulcus.
answer
a. coronary sinus.
question
The appropriate dosing regimen for epinephrine in cardiac arrest is: a. 1 mL of a 1:10,000 solution every 3 to 5 minutes. b. 0.1 mg/kg of a 1:10,000 solution every 3 minutes. c. 10 mL of a 1:1,000 solution every 3 to 5 minutes. d. 1 mg of a 1:10,000 solution every 3 to 5 minutes.
answer
d. 1 mg of a 1:10,000 solution every 3 to 5 minutes.
question
If you are unsure whether a conscious cardiac patient requires immediate transport versus continued assessment at the scene, you should: a. continue with the focused assessment to obtain more information. b. take the patient's vital signs and then make your transport decision. c. begin immediate transport and perform a focused assessment en route. d. contact medical control and inform him or her of the patient's condition.
answer
a. continue with the focused assessment to obtain more information.
question
A woman found her 48-year-old husband semiconscious on the couch. As she is escorting you to the patient, she tells you that he had an episode of chest pain the day before but refused to go to the hospital. The patient is responsive to pain only and is markedly diaphoretic. His blood pressure is 70/50 mm Hg, pulse is 140 beats/min and thready, and respirations are 28 breaths/min and shallow. The cardiac monitor reveals sinus tachycardia in lead II and a 12-lead ECG reveals signs of myocardial injury. You should: a. start an IV line, administer 5 mg of midazolam, intubate the patient's trachea, ventilate him at a rate of 15 breaths/min, begin transport, and start a dopamine infusion at 5 µg/kg/min en route to the hospital. b. keep the patient in a supine position, insert a nasal airway, assist his ventilations with a bag-mask device, begin transport, establish vascular access en route, consider a 100- to 200-mL saline bolus, and start an infusion of dopamine. c. place the patient in a semi-Fowler position to facilitate breathing, administer oxygen via nonrebreathing mask, begin transport, establish vascular access en route, and administer 20 mL/kg fluid boluses to improve his blood pressure. d. elevate the patient's legs, ventilate him with a bag-mask device, begin transport, establish vascular access en route, administer 6 mg of adenosine to slow his heart rate, and begin an infusion of epinephrine to increase his blood pressure.
answer
b. keep the patient in a supine position, insert a nasal airway, assist his ventilations with a bag-mask device, begin transport, establish vascular access en route, consider a 100- to 200-mL saline bolus, and start an infusion of dopamine.
question
A hypertensive emergency is MOST accurately defined as: a. an increase in the blood pressure due to medication noncompliance. b. an elevated blood pressure that is accompanied by a frontal headache. c. a blood pressure greater than 170/90 mm Hg with a severe nosebleed. d. an acute elevation in blood pressure with signs of end-organ damage.
answer
d. an acute elevation in blood pressure with signs of end-organ damage.
question
a. an increase in the blood pressure due to medication noncompliance. b. an elevated blood pressure that is accompanied by a frontal headache. c. a blood pressure greater than 170/90 mm Hg with a severe nosebleed. d. an acute elevation in blood pressure with signs of end-organ damage.
answer
d. an acute elevation in blood pressure with signs of end-organ damage.
question
Acute coronary syndrome (ACS) is a term used to describe: a. heart only. b. heart and blood vessels. c. blood vessels and lungs. d. heart, lungs, and blood vessels.
answer
d. heart, lungs, and blood vessels.
question
The sinoatrial node: a. cannot depolarize faster than 100 times/min. b. will outpace any slower conduction tissue. c. functions as the heart's secondary pacemaker. d. has an intrinsic firing rate of 40 to 60 beats per minute.
answer
b. will outpace any slower conduction tissue.
question
The duration of the QRS complex should be less than: a. 0.10 seconds. b. 0.12 seconds. c. 0.14 seconds. d. 0.16 seconds.
answer
b. 0.12 seconds.
question
A wide QRS complex that is preceded by a normal P wave indicates: a. that the rhythm is ventricular in origin. b. rapid conduction through the ventricles. c. a delay in conduction at the AV junction. d. an abnormality in ventricular conduction.
answer
d. an abnormality in ventricular conduction.
question
A bedridden patient with right-sided heart failure would be LEAST likely to present with: a. edema to the feet. b. presacral edema. c. hepatomegaly. d. jugular venous distention.
answer
a. edema to the feet.
question
Atherosclerosis is a process in which: a. the outer wall of a coronary artery becomes lined with masses of fatty tissue. b. calcium precipitates into the arterial walls, greatly reducing the artery's elasticity. c. plaque infiltrates the arterial wall, decreasing its elasticity and narrowing its lumen. d. plaque ruptures from a distant location and lodges in one of the coronary arteries.
answer
c. plaque infiltrates the arterial wall, decreasing its elasticity and narrowing its lumen.
question
In order to call a cardiac rhythm \"paroxysmal\" supraventricular tachycardia, you would have to: a. witness its onset and/or spontaneous termination. b. confirm the pacemaker origin with a 12-lead ECG. c. observe a consistent heart rate greater than 150 beats/min. d. ask the patient when he or she began feeling palpitations.
answer
a. witness its onset and/or spontaneous termination.
question
The blood vessel that branches off the right coronary artery and supplies blood to part of the conduction system is: a. circumflex artery. b. left coronary artery. c. anterior descending artery. d. posterior descending artery.
answer
d. posterior descending artery.
question
According to the Einthoven triangle, lead II is assessed by placing the: a. negative lead on the left arm and the positive lead on the left leg. b. positive lead on the left leg and the negative lead on the right arm. c. positive lead on the left arm and the negative lead on the right arm. d. negative lead on the right arm and the positive lead on the left leg.
answer
d. negative lead on the right arm and the positive lead on the left leg.
question
After administering 40 units of vasopressin to an elderly woman in bradycardic PEA, you should: a. pause CPR for no longer than 10 seconds and assess for a pulse. b. continue CPR and flush the IV line with 20 mL of normal saline. c. administer 1 mg of epinephrine 1:10,000 while CPR is ongoing. d. give naloxone to rule out opiate overdose as the cause of her arrest.
answer
b. continue CPR and flush the IV line with 20 mL of normal saline.
question
Approximately 80% of ventricular filling occurs: a. during systole. b. during diastole. c. when the semilunar valves are open. d. when the atrioventricular valves close.
answer
b. during diastole.
question
Which of the following drugs possesses beta-2-specific properties? a. Dopamine b. Levophed c. Proventil d. Epinephrine
answer
c. Proventil
question
Following 2 minutes of CPR, you reassess an unresponsive man's pulse and cardiac rhythm. He remains pulseless and the monitor displays coarse ventricular fibrillation. You should: a. continue CPR and intubate his trachea. b. resume CPR as the defibrillator is charging. c. perform 2 minutes of CPR and then reassess. d. continue CPR and establish IV or IO access.
answer
b. resume CPR as the defibrillator is charging.
question
Which of the following statements regarding second-degree heart block is MOST correct? a. Most second-degree heart blocks are transient in nature and resolve in the prehospital setting without the need for intervention in the emergency department. b. Second-degree heart block occurs when an impulse reaching the AV node is occasionally prevented from proceeding to the ventricles and causing a QRS complex. c. More than half of all second-degree heart blocks cause hemodynamic compromise and require transcutaneous cardiac pacing in the prehospital or hospital setting. d. Second-degree heart block is characterized by inconsistent P-R intervals, a QRS complex greater than 0.12 seconds, and a ventricular rate less than 40 beats/min.
answer
b. Second-degree heart block occurs when an impulse reaching the AV node is occasionally prevented from proceeding to the ventricles and causing a QRS complex.
question
Patients experiencing a right ventricular infarction: a. may present with hypotension. b. should not be given baby aspirin. c. often require higher doses of morphine. d. usually have anterior myocardial damage.
answer
a. may present with hypotension.
question
Leads V1 to V3 allow you to view the ________ wall of the left ventricle. a. septal b. lateral c. anterior d. anteroseptal
answer
d. anteroseptal
question
A 67-year-old man presents with severe dyspnea, coarse crackles to all lung fields, and anxiety. He has a history of several myocardial infarctions and long-standing hypertension. Which of the following interventions will have the MOST immediate and positive effect? a. Positive end-expiratory pressure ventilation b. IV or IO access and 20 to 40 mg of furosemide c. 0.4 mg sublingual nitroglycerin, up to 3 doses d. Supplemental oxygen via nonrebreathing mask
answer
a. Positive end-expiratory pressure ventilation
question
When administering aspirin to a patient with an acute coronary syndrome, you should: a. first check to make sure the patient is not overtly hypertensive. b. administer half the usual dose if the patient has a history of stroke. c. have him or her chew and swallow 160 to 325 mg of baby aspirin. d. give up to 325 mg of enteric-coated aspirin for the patient to swallow.
answer
c. have him or her chew and swallow 160 to 325 mg of baby aspirin.
question
An accelerated idioventricular rhythm is characterized by all of the following, EXCEPT: a. QRS complexes greater than 0.12 seconds in duration. b. irregular R-R intervals and a rate less than 40 beats/min. c. wide QRS complexes with P waves buried in the T waves. d. regular R-R intervals and a rate between 40 and 100 beats/min.
answer
b. irregular R-R intervals and a rate less than 40 beats/min.
question
Patients who are experiencing an infarction of the right ventricle: a. should not be given IV fluid boluses. b. often require high doses of nitroglycerin. c. are usually hypertensive and tachycardic. d. may present with significant hypotension.
answer
d. may present with significant hypotension.
question
What is the approximate maximum dose of lidocaine for a 200-pound patient? a. 275 mg b. 300 mg c. 325 mg d. 350 mg
answer
a. 275 mg
question
A wandering atrial pacemaker: a. has consistent P wave shapes. b. is generally faster than 100 beats/min. c. may have variable P-R intervals. d. is generally treated with atropine.
answer
c. may have variable P-R intervals.
question
Stable angina: a. typically subsides within 10 to 15 minutes. b. occurs after a predictable amount of exertion. c. usually requires both rest and nitroglycerin to subside. d. is characterized by sharp chest pain rather than pressure.
answer
b. occurs after a predictable amount of exertion.
question
Common complaints in patients experiencing an acute coronary syndrome (ACS) include all of the following, EXCEPT: a. fatigue. b. headache. c. chest pain. d. palpitations.
answer
b. headache.
question
Pericardial tamponade can be differentiated from a tension pneumothorax by the presence of: a. jugular venous distention. b. a narrowing pulse pressure. c. clear and equal breath sounds. d. alterations in the QRS amplitude.
answer
c. clear and equal breath sounds.
question
The MOST effective drug for the treatment of non-vagal-induced bradycardia is: a. atropine. b. dopamine. c. epinephrine. d. metoprolol.
answer
c. epinephrine.
question
Which of the following patients would MOST likely present with atypical signs and symptoms of an acute myocardial infarction? a. 49-year-old obese man b. 58-year-old diabetic woman c. 60-year-old man with anxiety d. 71-year-old woman with hypertension
answer
b. 58-year-old diabetic woman
question
The myocardial cells are stimulated, facilitating the action potential, when which ion rushed into the cell? a. potassium b. calcium c. chloride d. sodium
answer
d. sodium
question
An early complex that breaks the regularity of the underlying rhythm, and is characterized by a narrow QRS complex and an upright P wave that differs in shape and size from the P waves of the other complexes, MOST accurately describes a/an: a. atrial escape complex. b. wandering atrial pacemaker. c. junctional escape complex. d. premature atrial complex.
answer
d. premature atrial complex.
question
Spironolactone is a/an: a. beta blocker. b. vasodilator. c. diuretic. d. antiarrhythmic.
answer
c. diuretic.
question
To confirm ischemia or injury to the heart, you must see evidence in ___ or more contiguous leads. a. 2 b. 3 c. 4 d. 5
answer
a. 2
question
Monomorphic ventricular tachycardia: a. is characterized by QRS complexes that vary in size. b. presents with wide QRS complexes of a common shape. c. is treated as ventricular fibrillation if a pulse is present. d. is often irregular with occasional nonconducted P waves.
answer
b. presents with wide QRS complexes of a common shape
question
A right ventricular infarction is characterized by: a. ST segment elevation greater than 1 mm in lead V5R and ST segment depression in leads II, III, and aVF. b. ST segment elevation greater than 1 mm in lead V4R and ST segment elevation in leads II, III, and aVF. c. ST segment depression greater than 2 mm in lead V4R and ST segment elevation in leads II, III, and aVF. d. ST segment elevation greater than 2 mm in lead V5R and ST segment elevation in leads II, III, and aVF.
answer
b. ST segment elevation greater than 1 mm in lead V4R and ST segment elevation in leads II, III, and aVF.
question
In the context of cardiac compromise, syncope occurs due to: a. an increase in vagal tone. b. a drop in cerebral perfusion. c. a sudden cardiac dysrhythmia. d. an acute increase in heart rate.
answer
b. a drop in cerebral perfusion.
question
A 56-year-old man complains of chest tightness, shortness of breath, and nausea. During your assessment, you note that he appears confused. He is profusely diaphoretic, and has a blood pressure of 98/68 mm Hg and a rapid radial pulse. The cardiac monitor reveals a wide QRS complex tachycardia at a rate of 200 beats/min. After administering high-flow oxygen, you should: a. attempt to slow his heart rate with vagal maneuvers and then start an IV line. b. establish IV access, consider sedation, and perform synchronized cardioversion. c. establish vascular access and administer 150 mg of amiodarone over 10 minutes. d. obtain a 12-lead ECG tracing to determine the origin of his tachycardic rhythm.
answer
b. establish IV access, consider sedation, and perform synchronized cardioversion.
question
Hypertensive disease is characterized by: a. persistent elevation of the diastolic pressure. b. a diastolic blood pressure above 90 mm Hg. c. constant fluctuation in the systolic blood pressure. d. a systolic blood pressure greater than 140 mm Hg.
answer
a. persistent elevation of the diastolic pressure.
question
The recommended first-line treatment for third-degree heart block associated with bradycardia is: a. atropine sulfate. b. a dopamine infusion. c. an epinephrine infusion. d. transcutaneous pacing.
answer
d. transcutaneous pacing.
question
Once an advanced airway device has been inserted into a cardiac arrest patient: a. you should deliver one breath every 5 to 6 seconds. b. ventilations are delivered at a rate of 8 to 10 breaths/min. c. the compressor should pause so ventilations can be given. d. chest compressions should be increased to 120 per minute.
answer
b. ventilations are delivered at a rate of 8 to 10 breaths/min.
question
Special fibers known as intercalated discs, which speed electrical impulses from one muscle fiber to the next are found in the: a. uterine muscle. b. myocardium. c. gastrocnemius muscle. d. rectus femoris muscle.
answer
b. myocardium.
question
All of the following medications are angiotensin-converting enzyme (ACE) inhibitors, EXCEPT: a. Calan. b. Zestril. c. Univasc. d. Monopril.
answer
a. Calan.
question
A loud S3 heart sound, when heard in older adults, often signifies: a. emphysema. b. valve rupture. c. heart failure. d. pulmonary hypertension.
answer
c. heart failure.
question
Fibrinolytic medications are beneficial to certain patients with an acute myocardial infarction because they: a. decrease circulating platelets and thin the blood. b. convert plasminogen to plasmin and destroy a clot. c. destroy a clot by releasing fibrin into the bloodstream. d. break down the plasmin concentration inside a blood clot.
answer
b. convert plasminogen to plasmin and destroy a clot.
question
The Q-T interval would MOST likely be prolonged in patients: a. who take digitalis. b. who are hypocalcemic. c. with a rapid heart rate. d. who are hypercalcemic.
answer
b. who are hypocalcemic.
question
Cardiac output is influenced by: a. heart rate. b. stroke volume. c. heart rate and/or stroke volume. d. ejection fraction and heart rate.
answer
c. heart rate and/or stroke volume.
question
Which of the following clinical findings is LEAST suggestive of a peripheral vascular disorder? a. A bruit heard over the carotid artery b. Pain in the calf muscle while walking c. Swelling and pain along the course of a vein d. An S3 sound during auscultation of the heart
answer
d. An S3 sound during auscultation of the heart
question
When monitoring a patient's cardiac rhythm, it is MOST important to remember that: a. a heart rate below 60 beats per minute must be treated immediately. b. many patients with acute myocardial infarction experience asystole. c. the ECG does not provide data regarding the patient's cardiac output. d. the presence of a QRS complex correlates with the patient's pulse.
answer
c. the ECG does not provide data regarding the patient's cardiac output.
question
A pure alpha agent: a. causes marked vasoconstriction. b. has a direct effect on the heart rate. c. causes moderate bronchoconstriction. d. decreases the blood pressure by dilating the vessels.
answer
a. causes marked vasoconstriction.
question
On the ECG strip, a third-degree AV block usually appears as a: a. wide-QRS complex rhythm with a rate between 50 and 70 beats/min. b. slow, narrow-QRS complex rhythm with irregular P-P intervals. c. slow, wide-QRS complex rhythm with inconsistent P-R intervals. d. narrow-QRS complex rhythm with a rate less than 60 beats/min.
answer
c. slow, wide-QRS complex rhythm with inconsistent P-R intervals.
question
The MOST common cause of cardiac arrest in adult patients is: a. acute myocardial infarction. b. electrocution. c. an arrhythmia. d. respiratory failure.
answer
c. an arrhythmia.
question
Which of the following electrolytes maintains the depolarization phase? a. Sodium b. Calcium c. Potassium d. Magnesium
answer
b. Calcium
question
When assessing lead II via the limb leads, the negative lead should be placed on the: a. left arm. b. left leg. c. right arm. d. right leg.
answer
c. right arm.
question
If the R-R interval spans more than ____ large boxes on the ECG graph paper, the rate is less than 60 beats/min. a. 5 b. 6 c. 7 d. 8
answer
a. 5
question
The farther removed the conduction tissue is from the sinatrial node: a. the slower its intrinsic rate of firing. b. the longer the P-R interval will be. c. the faster its intrinsic rate of firing. d. the narrower the QRS complex will be.
answer
a. the slower its intrinsic rate of firing.
question
The area of conduction tissue in which electrical activity arises at any given time is called the: a. myocyte. b. pacemaker. c. sinus node. d. bundle of His.
answer
b. pacemaker.
question
You respond to the scene of an assault, where a 20-year-old man was struck in the chest with a steel pipe. Your assessment reveals that the patient is unresponsive, apneic, and pulseless. The MOST appropriate next intervention is to: a. perform 5 cycles of well-coordinated CPR. b. look for evidence of a pericardial tamponade. c. immediately assess the patient's cardiac rhythm. d. give 2 minutes of 15 compressions and 2 breaths.
answer
a. perform 5 cycles of well-coordinated CPR.
question
Percutaneous coronary interventions (PCI) involve: a. recanalizing a blocked coronary artery by passing a balloon or stent through a catheter via a peripheral artery. b. passing a 2-mm catheter through the femoral artery and administering a fibrinolytic agent through the catheter. c. using a large vein from one of the lower extremities to reroute blood flow past an occluded coronary artery. d. passing a guide wire through one of the external jugular veins to directly visualize an occluded coronary artery.
answer
a. recanalizing a blocked coronary artery by passing a balloon or stent through a catheter via a peripheral artery.
question
A major complication associated with atrial fibrillation is: a. clot formation in the fibrillating atria. b. a significant reduction in atrial filling. c. pulmonary congestion and hypoxemia. d. a profound increase in the atrial kick.
answer
a. clot formation in the fibrillating atria.
question
The atrioventricular (AV) junction: a. includes the AV node but not the bundle of His. b. is the dominant and fastest pacemaker in the heart c. receives its blood supply from the circumflex artery. d. is composed of the AV node and surrounding tissue.
answer
d. is composed of the AV node and surrounding tissue.
question
A patient with a medical condition that requires antiplatelet therapy would MOST likely be taking: a. Coreg b. Altace c. Zocor d. Plavix
answer
d. Plavix
question
A 33-year-old woman presents with an acute onset of \"fluttering\" in her chest. She is conscious and alert but is somewhat anxious. She denies any significant medical problems, but states that she has been under a lot of stress at work. You apply the cardiac monitor, which reveals a narrow QRS complex tachycardia at a rate of 170 beats/min. The patient's blood pressure is 140/90 mm Hg and she is breathing without difficulty. The MOST appropriate treatment for this patient involves: a. oxygen, vagal maneuvers, and emotional support. b. vagal maneuvers, IV access, and 0.25 mg/kg of diltiazem. c. oxygen, emotional support and 2.5 mg of midazolam IM. d. oxygen, IV access, vagal maneuvers, and 6 mg of adenosine.
answer
d. oxygen, IV access, vagal maneuvers, and 6 mg of adenosine.
question
Abnormal neurologic signs that accompany hypertensive encephalopathy occur when: a. pressure in the brain causes transient dysfunction of the parietal lobe and cerebral vasodilation. b. the mean arterial pressure exceeds 100 mm Hg and blood is forced from the brain and into the spinal cord. c. neurons sustain permanent damage secondary to a single increase in blood pressure above 200/130 mm Hg. d. pressure causes a breach in the blood brain barrier and fluid leaks out, causing an increase in intracranial pressure.
answer
d. pressure causes a breach in the blood brain barrier and fluid leaks out, causing an increase in intracranial pressure.
question
If a particular interval on the ECG graph paper is 1.5 small boxes in width, the interval would be measured as: a. 0.06 seconds. b. 2 millimeters. c. 45 milliseconds. d. 60 milliseconds.
answer
a. 0.06 seconds.
question
Ventricular bigeminy occurs when: a. two PVCs occur in a row. b. every second complex is a PVC. c. at least two differently shaped PVCs occur. d. a 6-second strip contains at least two PVCs.
answer
b. every second complex is a PVC.
question
Death in the prehospital setting following an acute myocardial infarction is MOST often the result of: a. asystole. b. myocardial rupture. c. cardiogenic shock. d. ventricular fibrillation.
answer
d. ventricular fibrillation.
question
In contrast to a patient with asthma, a patient with left-sided heart failure: a. presents with a dry, nonproductive cough and diffuse wheezing in all lung fields. b. experiences acute weight gain and takes medications such as digoxin and a diuretic. c. presents with a hyperinflated chest, use of accessory muscles, and expiratory wheezing. d. is typically a younger patient with a history of a recent upper respiratory infection.
answer
b. experiences acute weight gain and takes medications such as digoxin and a diuretic.
question
A beta-blocking drug would counteract all of the following medications, EXCEPT: a. atropine. b. epinephrine. c. isoproterenol. d. norepinephrine.
answer
a. atropine.
question
A patient in cardiogenic shock without cardiac arrhythmias will benefit MOST from: a. supplemental oxygen. b. a high-dose vasopressor infusion. c. a 250 mL bolus of a crystalloid solution. d. rapid transport to an appropriate hospital.
answer
d. rapid transport to an appropriate hospital.
question
The __________ represents the end of ventricular depolarization and the beginning of repolarization. a. J point b. T wave c. ST segment d. T-P interval
answer
a. J point
question
When analyzing a cardiac rhythm strip in lead II, you should routinely evaluate all of the following components, EXCEPT the: a. QRS width. b. P-R interval. c. S-T segment. d. R-R interval.
answer
c. S-T segment.
question
If the ECG leads are applied correctly, the PQRST configuration should be inverted in lead: a. I. b. II. c. aVR. d. aVL.
answer
c. aVR.
question
What is the R-on-T phenomenon? a. A PVC that occurs when the ventricles are not fully repolarized b. When the R wave occurs at the J point of the next cardiac cycle c. A unifocal PVC that occurs during the upslope of any given T wave d. A PVC that occurs during a time when the ventricles are depolarizing
answer
a. A PVC that occurs when the ventricles are not fully repolarized
question
A 17-year-old man complains of palpitations and lightheadedness that began suddenly about 20 minutes ago. His blood pressure is 118/74 mm Hg, heart rate is rapid and regular, and respirations are 18 breaths/min. The cardiac monitor reveals a narrow QRS complex tachycardia at 180 beats/min. As you are applying supplemental oxygen, the cardiac rhythm spontaneously converts to a sinus rhythm. Closer evaluation of his rhythm reveals a rapid up slope to the R wave immediately after the end of the P wave. Which of the following statements regarding this scenario is MOST correct? a. This patient likely has Wolff-Parkinson-White syndrome. b. The patient's ECG abnormality is called an Osborn wave. c. The ECG abnormality is caused by failure of the AV node. d. In this patient, there is a delay in ventricular depolarization.
answer
a. This patient likely has Wolff-Parkinson-White syndrome.
question
During the refractory period: a. the heart is in a state of partial repolarization. b. the heart is partially charged, but cannot contract. c. the cell is depolarized or in the process of repolarizing. d. the heart muscle is depleted of energy and needs to recharge.
answer
c. the cell is depolarized or in the process of repolarizing.
question
Unlike the parasympathetic nervous system, the sympathetic nervous system: a. is not under the direct control of the autonomic nervous system. b. provides a mechanism for the body to adapt to changing demands. c. is blocked when drugs such as atropine are administered. d. constricts the pupils and increases gastrointestinal function when stimulated.
answer
b. provides a mechanism for the body to adapt to changing demands.
question
A 68-year-old woman presents with an acute onset of confusion, shortness of breath, and diaphoresis. Her blood pressure is 72/50 mm Hg, her heart rate is slow and weak, and her respirations are increased and shallow. The ECG reveals a third-degree heart block at a rate of 38 beats/min. After placing the patient on high-flow oxygen, you should: a. start an IV and administer 0.5 mg atropine. b. obtain a 12-lead ECG to detect an acute MI. c. obtain vascular access and give a fluid bolus. d. immediately attempt transcutaneous pacing.
answer
d. immediately attempt transcutaneous pacing.
question
If an impulse generated by the AV node begins moving upward through the atria before the other part of it enters the ventricles: a. the P-R intervals will be greater than 0.20 seconds. b. an upright P wave will appear after the QRS complex. c. an inverted P wave will appear before the QRS complex. d. a small inverted P wave will be buried in the QRS complex.
answer
c. an inverted P wave will appear before the QRS complex.
question
When assessing an anxious patient who presents with tachycardia, you must: a. obtain a 12-lead ECG tracing before initiating any treatment. b. determine if the tachycardia is causing hemodynamic instability. c. prepare for cardioversion if the rate is less than 150 beats/min. d. administer diazepam or midazolam to facilitate your assessment.
answer
b. determine if the tachycardia is causing hemodynamic instability.
question
In contrast to arteries, veins: a. do not contain valves that prevent backflow of blood. b. have less capacity to increase the size of their diameter. c. are more likely to distend when exposed to backpressure. d. operate on the high-pressure side of the circulatory system.
answer
c. are more likely to distend when exposed to backpressure.
question
The brief pause between the P wave and QRS complex represents: a. depolarization of the inferior part of the atria. b. the period of time when the atria are repolarizing. c. full dispersal of electricity throughout both atria. d. a momentary conduction delay at the AV junction.
answer
d. a momentary conduction delay at the AV junction.
question
The heart's anatomic location is MOST accurately described as being: a. retrosternal. b. hemithoracic. c. submediastinal. d. supradiaphragmatic.
answer
a. retrosternal
question
Deoxygenated blood from the head and upper extremities empties into the right atrium from which vessel? a. superior vena cava b. inferior vena cava c. pulmonary artery d. pulmonary vein
answer
a. superior vena cava
question
What physiologic effect occurs within the first 5 to 10 minutes after administering furosemide (Lasix)? a. Increased preload b. Peripheral venous pooling c. Excretion of water by the kidneys
answer
b. Peripheral venous pooling
question
Which of the following statements regarding treatment for a first-degree heart block is MOST correct? a. Treatment is generally not indicated unless the rate is slow and cardiac output is impaired. b. Most first-degree heart blocks are associated with significant bradycardia and require atropine. c. First-degree heart block is often accompanied by a compensatory tachycardia that requires treatment. d. Transcutaneous cardiac pacing should be initiated without delay for patients with a first-degree heart block.
answer
a. Treatment is generally not indicated unless the rate is slow and cardiac output is impaired.
question
The firing of an artificial ventricular pacemaker causes: a. a change in the shape of the preceding P waves. b. a vertical spike followed by a wide QRS complex. c. a small spike followed by a narrow QRS complex. d. a wide QRS complex followed by a vertical spike.
answer
b. a vertical spike followed by a wide QRS complex
question
Hypertension is present when the blood pressure: a. increases by 20 mm Hg above a person's normal blood pressure. b. is consistently greater than 140/90 mm Hg while at rest. c. is above 160 mm Hg systolic during strenuous exertion. d. rises acutely during an emotionally stressful situation.
answer
b. is consistently greater than 140/90 mm Hg while at rest.
question
Which of the following pulseless rhythms is NOT treated as pulseless electrical activity (PEA)? a. Sinus bradycardia b. Idioventricular rhythm c. Ventricular tachycardia d. Junctional escape rhythm
answer
c. Ventricular tachycardia
question
Cholinesterase is a naturally occurring chemical that: a. increases epinephrine production. b. regulates acetylcholine in the body. c. stimulates activity of the vagus nerve. d. causes a natural slowing of the heart rate.
answer
b. regulates acetylcholine in the body.
question
Untreated ventricular tachycardia would MOST likely deteriorate to: a. asystole. b. torsade de pointes. c. ventricular fibrillation. d. pulseless electrical activity.
answer
c. ventricular fibrillation.
question
Patients with a heart rate greater than 150 beats/min usually become unstable because of: a. reduced ventricular filling. b. an increase in the atrial kick. c. increased right atrial preload. d. a significantly reduced afterload.
answer
a. reduced ventricular filling.
question
The circumflex branch of the left coronary artery supplies the _________ wall of the left ventricle. a. septal b. lateral c. anterior d. inferior
answer
b. lateral
question
Which of the following medications is a calcium channel blocker? a. Lanoxin b. Cardizem c. Tenormin d. Capoten
answer
b. Cardizem
question
Normally, the S-T segment should be: a. at the level of the isoelectric line. b. elevated by no more than 1 mm. c. depressed by no more than 2 -mm. d. invisible on a normal ECG tracing.
answer
a. at the level of the isoelectric line.
question
Depolarization, the process by which muscle fibers are stimulated to contract, occurs when: a. cell wall permeability changes and sodium rushes into the cell. b. calcium ions rapidly enter the cell, facilitating contraction. c. potassium ions escape from the cell through specialized channels. d. cardiac muscle relaxes in response to a cellular influx of calcium.
answer
a. cell wall permeability changes and sodium rushes into the cell.
question
How many large boxes on the ECG graph paper represent 6 seconds? a. 20 b. 30 c. 40 d. 50
answer
b. 30
question
You and an EMT-B are performing CPR on an elderly woman in cardiac arrest as your paramedic partner prepares to intubate her. After the patient has been intubated and proper ET tube placement has been confirmed, you should: a. perform asynchronous CPR while ventilating the patient at a rate of 8 to 10 breaths/min. b. instruct the EMT-B to pause after 30 compressions so your partner can deliver 2 ventilations. c. administer 2.5 mg of epinephrine via the ET tube and hyperventilate the patient to ensure drug dispersal. d. direct your partner to deliver 1 breath every 3 to 5 seconds as the EMT-B continues chest compressions.
answer
a. perform asynchronous CPR while ventilating the patient at a rate of 8 to 10 breaths/min.
question
Shortly after administering a second dose of 4 mg of morphine to a 49-year-old woman who is experiencing chest pain, the patient's level of consciousness markedly decreases. Further assessment reveals that she is hypotensive, bradycardic, and hypoventilating. You should: a. administer 0.5 mg of atropine and reassess her. b. assist her ventilations and administer naloxone. c. elevate her legs and give a 500 mL saline bolus. d. immediately intubate her to protect her airway.
answer
b. assist her ventilations and administer naloxone.
question
Disruption of blood flow into the left common carotid artery would MOST likely present with signs and symptoms of a/an: a. ischemic stroke. b. pericardial tamponade. c. hemorrhagic stroke. d. myocardial infarction.
answer
a. ischemic stroke.
question
A 55-year-old man complains of severe pain between his shoulder blades, which he describes as \"ripping\" in nature. He tells you that the pain began suddenly and has been intense and unrelenting since its onset. His medical history includes hypertension; however, he admits to being noncompliant with his antihypertensive medication. Which of the following assessment findings would MOST likely reinforce your suspicion regarding the cause of his pain? a. Disappearance of radial pulses during inspiration b. Difference in blood pressure between the two arms c. ST segment depression on the 12-lead ECG tracing d. Bruits to both carotid arteries during auscultation
answer
b. Difference in blood pressure between the two arms
question
You are dispatched to a grocery store for a 39-year-old woman with a severe headache. The patient advises you that her headache, which was present when she woke up this morning, is located in the back of her head. She is conscious and alert, with a blood pressure of 194/112 mm Hg, pulse of 100 beats/min and strong, and respirations of 14 breaths/min and regular. She denies a history of hypertension or any other significant medical problems. The closest appropriate facility is located 15 miles away. You should: a. administer supplemental oxygen, start an IV line of normal saline at a keep-open rate, and transport. b. start an IV line of normal saline, give her 0.4 mg of sublingual nitroglycerin, and transport at once. c. give high-flow oxygen, establish vascular access, begin transport, and administer labetalol en route. d. administer oxygen as tolerated, give up to 5 mg of morphine intramuscularly, and transport promptly.
answer
a. administer supplemental oxygen, start an IV line of normal saline at a keep-open rate, and transport.
question
Which of the following factors would present the GREATEST difficulty when distinguishing supraventricular tachycardia from ventricular tachycardia? a. Aberrant conduction b. Absence of P waves c. Retrograde conduction d. The rate of the rhythm
answer
a. Aberrant conduction
question
On the ECG graph paper, height is measured in _____________ and width is measure in ____________. a. centimeters, seconds b. milliseconds, millimeters c. seconds, centimeters d. millimeters, milliseconds
answer
d. millimeters, milliseconds
question
Which of the following statements regarding sinus bradycardia is MOST correct? a. Treatment focuses on the patient's tolerance to the bradycardia. b. Symptomatic bradycardia is often caused by a decreased atrial rate. c. Sinus bradycardia often requires multiple doses of atropine to correct it. d. Sinus bradycardia is caused by decreased vagal tone in most patients.
answer
a. Treatment focuses on the patient's tolerance to the bradycardia
question
What prevents the backflow of blood during ventricular contraction? a. The aortic valve b. Semilunar valves c. The pulmonic valve d. Atrioventricular valves
answer
d. Atrioventricular valves
question
Damage to the cardiac electrical conduction system caused by an acute myocardial infarction MOST commonly results in: a. severe tachycardia. b. ventricular arrhythmias. c. acute bundle branch block. d. bradycardia or heart block.
answer
d. bradycardia or heart block.
question
Cardiac arrhythmias following an acute myocardial infarction: a. tend to originate from ischemic areas around the infarction. b. typically manifest as atrial fibrillation or atrial tachycardia. c. generally originate from the center of the infarcted tissues. d. are uncommon within the first 24 hours after the infarction.
answer
a. tend to originate from ischemic areas around the infarction
question
You are assessing the 12-lead tracing of a 40-year-old man with chest pain and note ST segment elevation in leads II, III, and aVF. Lead V4R shows 2-mm ST segment elevation. The patient's blood pressure is 88/58 mm Hg and his heart rate is 72 beats/min and regular. He denies any significant past medical history, but is allergic to salicylates. After placing the patient on oxygen and starting an IV line of normal saline, you should: a. administer up to 325 mg of baby aspirin. b. give 2 mg increments of morphine sulfate. c. start a dopamine infusion at 2 µg/kg/min. d. give crystalloid boluses to increase preload.
answer
d. give crystalloid boluses to increase preload.
question
A prolonged P-R interval: a. is greater than 0.12 seconds. b. suggests damage to the SA node. c. may indicate a diseased AV node. d. indicates that the AV node was bypassed.
answer
c. may indicate a diseased AV node.
question
You are assessing a conscious and alert middle-aged male who complains of chest discomfort and nausea. His blood pressure is 112/70 mm Hg, pulse is 90 beats/min and regular, and respirations are 20 breaths/min and regular. The patient's past medical history is significant for hypothyroidism and hyperlipidemia. His medications include Synthroid, Lipitor, Cialis, and 1 baby aspirin per day. Which of the following medications would you LEAST likely administer? a. Aspirin b. Fentanyl c. Morphine d. Nitroglycerin
answer
d. Nitroglycerin
question
Changes in cardiac contractility may be induced by medications that have a positive or negative ___________ effect. a. vasoactive b. dromotropic c. inotropic d. chronotropic
answer
c. inotropic
question
The inferior wall of the left ventricle is supplied by the: a. right coronary artery. b. left coronary artery. c. circumflex artery. d. left anterior descending artery.
answer
a. right coronary artery
question
Which of the following interventions should be performed en route to the hospital during a lengthy transport of a patient with a suspected myocardial infarction? a. Supplemental oxygen b. Aspirin administration c. IV therapy and analgesia d. 12-lead electrocardiography
answer
c. IV therapy and analgesia
question
Which of the following clinical findings is LEAST suggestive of left side heart failure? a. An S3 gallop b. Sacral edema c. Crackles in the lungs d. Shortness of breath
answer
b. Sacral edema
question
You have just performed synchronized cardioversion on a patient with unstable ventricular tachycardia. Upon reassessment, you note that the patient is unresponsive, apneic, and pulseless. You should: a. desynchronize the defibrillator, defibrillate one time, and check for a pulse. b. ensure that the synchronizer is off, defibrillate, and immediately begin CPR. c. increase the energy setting on the defibrillator and repeat the cardioversion. d. perform 5 cycles of CPR, reassess the cardiac rhythm, and defibrillate if needed.
answer
b. ensure that the synchronizer is off, defibrillate, and immediately begin CPR
question
You have just administered 0.4 mg of sublingual nitroglycerin to a 60-year-old woman with severe chest pain. The patient is receiving high-flow oxygen and has an IV line of normal saline in place. After 5 minutes, the patient states that the pain has not subsided. You should: a. repeat the nitroglycerin. b. reassess her blood pressure. c. give her a 250-mL saline bolus. d. administer 2 to 4 mg of morphine.
answer
b. reassess her blood pressure.
question
Which of the following differentiates an atrial rhythm from a sinus rhythm? a. Tachycardia b. Profound bradycardia c. Dissociated P waves d. Varying shapes in P waves
answer
d. Varying shapes in P waves
question
Potential contraindications to fibrinolysis include all of the following, EXCEPT: a. major trauma or surgery within the past 6 weeks. b. a history of structural central nervous system disease. c. a history of anaphylactic shock caused by salicylates. d. significant closed head trauma within the past 3 months.
answer
c. a history of anaphylactic shock caused by salicylates.
question
A patient with orthopnea: a. experiences dyspnea during periods of exertion. b. prefers a semisitting position to facilitate breathing. c. experiences worsened dyspnea while lying down. d. sleeps in a recliner due to severe right heart failure.
answer
c. experiences worsened dyspnea while lying down
question
A patient with cardiogenic pulmonary edema and shock should be positioned: a. in the Trendelenburg position. b. in a semi-Fowler's position. c. supine with the legs elevated. d. in a lateral recumbent position.
answer
b. in a semi-Fowler's position.
question
The MOST common symptom directly related to hypertension is: a. epistaxis. b. headache. c. dizziness. d. blurred vision.
answer
b. headache
question
When using limb leads, any impulse moving toward a positive electrode will: a. cause a positive deflection on the ECG. b. produce a significant amount of artifact. c. cause a negative deflection on the ECG. d. manifest with narrow QRS complexes.
answer
a. cause a positive deflection on the ECG.
question
Torsade de pointes: a. presents with wide QRS complexes that are all of the same shape, size, and vector direction. b. is a lethal ventricular rhythm that is usually caused by ingestion or injection of CNS-depressant drugs. c. is generally less serious than monomorphic ventricular tachycardia and is usually not treated in the field. d. is a variant of polymorphic ventricular tachycardia and is often caused by a prolonged Q-T interval.
answer
d. is a variant of polymorphic ventricular tachycardia and is often caused by a prolonged Q-T interval.
question
Anatomically contiguous leads view: a. opposite walls of the heart. b. only the lateral wall of the heart. c. the same general area of the heart. d. only the anterior wall of the heart.
answer
c. the same general area of the heart
question
Myocardial ischemia occurs when the heart muscle: a. is deprived of oxygen because of a blocked coronary artery. b. undergoes necrosis because of prolonged oxygen deprivation. c. suffers oxygen deprivation secondary to coronary vasodilation. d. experiences a decreased oxygen demand and an increased supply.
answer
a. is deprived of oxygen because of a blocked coronary artery.
question
A patient with an elevated cholesterol level would MOST likely take: a. Inderal. b. Altacor. c. Isordil. d. Diovan.
answer
b. Altacor.
question
Automaticity is MOST accurately defined as the ability of the heart to: a. generate an electrical impulse from the same site every time. b. spontaneously conduct an electrical impulse between cardiac cells. c. generate its own electrical impulses without stimulation from nerves. d. increase or decrease its heart rate based on the body's metabolic needs.
answer
c. generate its own electrical impulses without stimulation from nerves
question
The S1 heart sounds represent: a. closure of the mitral and tricuspid valves. b. the end of ventricular contraction. c. closure of the aortic and pulmonic valves. d. the beginning of atrial contraction.
answer
a. closure of the mitral and tricuspid valves.
question
Which of the following is NOT a modifiable risk factor for atherosclerosis? a. hypercholesterolemia b. hypertension c. diabetes d. obesity
answer
c. diabetes
question
Which of the following prescribed medications would a patient with chronic atrial fibrillation MOST likely take? a. Plavix and Vasotec b. Lisinopril and aspirin c. Digoxin and Coumadin d. Cordarone and furosemide
answer
c. Digoxin and Coumadin
question
The MOST common cause of right-sided heart failure is: a. left-sided heart failure. b. pulmonary hypotension. c. acute pulmonary embolism. d. long-standing emphysema.
answer
a. left-sided heart failure.
question
Which of the following statements MOST accurately describes an acute myocardial infarction (AMI)? a. Death of the myocardium secondary to spasm of a major coronary artery b. Injury to a portion of the heart muscle secondary to atherosclerotic disease c. Damage to the left ventricle following occlusion of the left coronary artery d. Necrosis of a portion of the myocardium due to a prolonged lack of oxygen
answer
d. Necrosis of a portion of the myocardium due to a prolonged lack of oxygen
question
Common causes of cardiac arrest include all of the following, EXCEPT: a. hypovolemia. b. hyperglycemia. c. cardiac tamponade. d. pulmonary embolism.
answer
b. hyperglycemia
question
Injury to or disease of the ______________ may cause prolapse of a cardiac valve leaflet, allowing blood to regurgitate from the ventricle into the atrium. a. coronary sulcus b. chordae tendineae c. interatrial septum d. coronary sinus
answer
b. chordae tendineae
question
When examining the chest of a patient who has an automated implanted cardioverter defibrillator (AICD), you would MOST likely find it: a. just below the xiphoid process. b. below the left or right clavicle. c. in the lower aspect of the chest. d. just lateral to the lower sternum.
answer
b. below the left or right clavicle.
question
Which of the following ECG waveforms represents ventricular depolarization? a. T wave b. ST segment c. QRS complex d. U wave
answer
c. QRS complex
question
The triple-layered sac which surrounds and protects the heart is the: a. epicardium. b. pericardium c. endocardium. d. myocardium.
answer
b. pericardium
question
A decreased cardiac output secondary to a heart rate greater than 150 beats/min is caused by: a. myocardial stretching due to increased preload. b. decreases in stroke volume and ventricular filling. c. increased automaticity of the cardiac pacemaker. d. ectopic pacemaker sites in the atria or ventricles.
answer
b. decreases in stroke volume and ventricular filling.
question
Treatment for a patient with symptomatic bradycardia includes: a. 1 mg of epinephrine 1:10,000. b. 1 mg of atropine via IV push. c. transcutaneous cardiac pacing. d. a dopamine infusion at 20 mg/min.
answer
c. transcutaneous cardiac pacing.
question
After delivering a shock to a patient in pulseless ventricular tachycardia, you should: a. resume CPR. b. check for a pulse. c. reassess the cardiac rhythm. d. deliver 2 effective ventilations.
answer
a. resume CPR.
question
Bombardment of the AV node by more than one impulse, potentially blocking the pathway for one impulse and allowing the other impulse to stimulate cardiac cells that have already depolarized, is called: a. fusion. b. reentry. c. ectopy. d. excitability.
answer
b. reentry.
question
Which of the following actions should NOT occur while CPR is in progress? a. Advanced airway insertion b. Cardiac rhythm assessment c. Assessing for a palpable pulse d. Establishment of vascular access
answer
b. Cardiac rhythm assessment
question
When performing CPR on an adult in cardiac arrest, it is important to: a. deliver at least 80 to 90 compressions per minute. b. limit interruptions in chest compressions to 20 seconds. c. deliver forceful ventilations between compressions. d. allow the chest to fully recoil between compressions.
answer
d. allow the chest to fully recoil between compressions.
question
A normal Q-T interval lasts: a. 0.15 to 0.25 seconds. b. 0.30 to 0.40 seconds. c. 0.36 to 0.44 seconds. d. 0.38 to 0.48 seconds.
answer
c. 0.36 to 0.44 seconds.
question
The presence of an apparent P wave at the end of the QRS complex is MOST consistent with: a. a delta wave. b. hyponatremia. c. hypercalcemia. d. hypothermia.
answer
d. hypothermia.
question
An increase in peripheral vascular resistance causes: a. an increase in afterload. b. a decrease in blood pressure. c. an increase in cardiac output. d. a decrease in cardiac workload.
answer
b. a decrease in blood pressure.
question
A classic sign of atrial flutter is: a. a constant 2:1 conduction ratio. b. the presence of sawtooth F waves. c. a ventricular rate less than 100 beats/min. d. an irregular and consistent R-R interval.
answer
b. the presence of sawtooth F waves.
question
Vasoconstriction occurs following stimulation of: a. beta-1 receptors. b. beta-2 receptors. c. alpha receptors. d. alpha and beta receptors.
answer
c. alpha receptors.
question
What is the MOST correct sequence of treatment for a patient with a suspected acute myocardial infarction? a. Oxygen, aspirin, nitroglycerin, morphine b. Oxygen, nitroglycerin, aspirin, morphine c. Aspirin, nitroglycerin, oxygen, morphine d. Morphine, oxygen, aspirin, nitroglycerin
answer
a. Oxygen, aspirin, nitroglycerin, morphine
question
Ischemia to the anterior wall of the myocardium would present with: a. T wave inversion in leads V3 and V4. b. ST segment depression in leads I and aVL. c. T wave inversion in leads II, III, and aVF. d. ST segment elevation in leads V3 and V4.
answer
a. T wave inversion in leads V3 and V4.
question
Which of the following statements is MOST correct? a. Lead I is contiguous with lead II. b. Lead II is contiguous with leads V6 and aVL. c. Lead V6 is contiguous with leads V4 and V5. d. Lead III is contiguous with leads II and aVF.
answer
d. Lead III is contiguous with leads II and aVF.
question
Lead I views the ________ side of the heart, while lead aVF views the _________ part of the heart. a. left, bottom b. top, right c. bottom, top d. right, left
answer
a. left, bottom
question
Paroxysmal nocturnal dyspnea (PND) is MOST accurately defined as: a. dyspnea that is brought on by excessive movement during sleep. b. sitting upright in a chair in order to facilitate effective breathing. c. the inability to function at night due to severe difficulty breathing. d. acute shortness of breath that suddenly awakens a person from sleep.
answer
d. acute shortness of breath that suddenly awakens a person from sleep.
question
In addition to prompt transport, the goal of prehospital management for a patient with a suspected aortic dissection includes: a. lowering the blood pressure. b. IV fluid boluses. c. high-flow oxygen. d. adequate pain relief.
answer
d. adequate pain relief.
question
Ventricular fibrillation occurs when: a. the ventricles quiver rather than contract normally, while organized atrial contractions continue as normal. b. the ventricles become the primary pacemaker for the heart, resulting in a rapid and irregular ventricular rhythm. c. many different cells in the heart depolarize independently rather than in response to an impulse from the SA node. d. cardiac cells in the ventricles fail to completely repolarize, resulting in a decrease in ventricular automaticity.
answer
c. many different cells in the heart depolarize independently rather than in response to an impulse from the SA node.
question
Which of the following medications has a direct blood-thinning effect? a. Plavix b. Aspirin c. Accupril d. Warfarin
answer
d. Warfarin
question
Which of the following blood pressure readings is MOST suggestive of a patient who has arteriosclerosis? a. 140/90 mm Hg b. 150/80 mm Hg c. 160/70 mm Hg d. 180/100 mm Hg.
answer
c. 160/70 mm Hg
question
Lowering of a significantly elevated blood pressure: a. is best accomplished in a hospital setting. b. is contraindicated in the prehospital setting. c. should be initiated in the field with labetalol. d. is routinely accomplished with nitroglycerin.
answer
a. is best accomplished in a hospital setting.
question
A key to interpreting a Mobitz type II second-degree heart block is to remember that: a. unlike a Mobitz type I second-degree heart block, a type II heart block is always regular. b. in this type of heart block, the P-R interval gets progressively longer until a P wave is not conducted. c. the P-R interval of all of the conducted P waves and their corresponding QRS complexes is constant. d. most type II second degree AV blocks have more than two nonconducted P waves that occur in succession.
answer
c. the P-R interval of all of the conducted P waves and their corresponding QRS complexes is constant
question
Unlike an idioventricular rhythm, an agonal rhythm: a. is associated with a faster rate. b. does not produce a palpable pulse. c. is associated with a lower mortality rate. d. indicates a regular ventricular pacemaker.
answer
b. does not produce a palpable pulse
question
The preferred antiarrhythmic medication and initial dose for a patient with refractory ventricular fibrillation or pulseless ventricular tachycardia is: a. lidocaine, 1.5 mg/kg. b. amiodarone, 300 mg. c. lidocaine, 0.75 mg/kg. d. procainamide, 20 mg/min.
answer
b. amiodarone, 300 mg.
question
In addition to supplemental oxygen, treatment of a patient with left-sided heart failure includes: a. a saline lock, a selective beta-2 adrenergic medication, and bicarbonate. b. an IV of normal saline, a 20 mL/kg fluid bolus, and a diuretic medication. c. a saline lock, fentanyl, and intubation facilitated by pharmacologic agents. d. an IV of normal saline to keep the vein open, nitroglycerin, and morphine.
answer
d. an IV of normal saline to keep the vein open, nitroglycerin, and morphine.
question
In contrast to the right side of the heart, the left side of the heart: a. drives blood out of the heart against the relatively high resistance of the systemic circulation. b. is a high-pressure pump that sends blood through the pulmonary circulation and to the lungs. c. is a relatively low-pressure pump that must stretch its walls in order to force blood through the aorta. d. drives blood out of the heart against the relatively low resistance of the pulmonary circulation.
answer
a. drives blood out of the heart against the relatively high resistance of the systemic circulation
question
A run of ventricular tachycardia occurs if at least ____ PVCs occur in a row. a. 2 b. 3 c. 4 d. 5
answer
b. 3
question
The ability of the heart to vary the degree of its contraction without stretching is called: a. contractility. b. chronotropy. c. automaticity. d. Frank-Starling effect.
answer
a. contractility.
question
A patient is generally considered a potential candidate for fibrinolytic therapy if he or she has experienced chest discomfort for less than ____ hours. a. 12 b. 16 c. 18 d. 24
answer
a. 12
question
Which of the following situations would contraindicate the administration of nitroglycerin? a. Hypersensitivity to salicylates b. Systolic BP less than 90 mm Hg c. Levitra use within the last 72 hours d. Use of Plavix within the last 12 hours
answer
b. Systolic BP less than 90 mm Hg
question
Epinephrine is used to treat patients in anaphylactic shock because of its effects of: a. vasodilation and bronchoconstriction. b. bronchodilation and vasoconstriction. c. increased heart rate and automaticity. d. parasympathetic nervous system blockade.
answer
b. bronchodilation and vasoconstriction.
question
Which of the following is NOT a goal for decreasing the risk of cardiovascular disease? a. Increasing aerobic exercise b. Controlling high blood pressure c. Increasing LDL cholesterol level d. Increasing HDL cholesterol level
answer
c. Increasing LDL cholesterol level
question
The left main coronary artery subdivides into the: a. left anterior ascending and descending arteries. b. left anterior descending and circumflex arteries. c. left posterior ascending and circumflex arteries. d. right coronary and left posterior descending arteries.
answer
b. left anterior descending and circumflex arteries.
question
The layers of the wall of the heart, beginning with the outermost layer, are the: a. epicardium, myocardium, and endocardium. b. endocardium, epicardium, and myocardium. c. myocardium, epicardium, and endocardium. d. epicardium, endocardium, and myocardium.
answer
a. epicardium, myocardium, and endocardium.
question
You are called to a local gym for a patient with nausea. Your patient, a 29-year-old man, tells you that he thinks he has a \"stomach bug.\" He is conscious and alert, denies chest pain or shortness of breath, and tells you that he has been nauseated for the last 4 hours, but has not vomited. His blood pressure is 124/66 mm Hg, pulse is 46 beats/min and strong, respirations are 20 breaths/min and regular, and room air oxygen saturation is 99%. The cardiac monitor reveals a sinus bradycardia. You should: a. give 100% oxygen, start two large-bore IV lines, administer 20 mL/kg normal saline boluses, and transport. b. administer oxygen via nasal cannula, start an IV of normal saline, consider administering an antiemetic, and transport. c. advise him that he can probably drive himself to the emergency department or schedule an appointment with his physician. d. apply high-flow oxygen via nonrebreathing mask, start an IV line, administer 0.5 mg of atropine, and transport to the closest facility.
answer
b. administer oxygen via nasal cannula, start an IV of normal saline, consider administering an antiemetic, and transport.
question
The effect on the velocity of electrical conduction is referred to as the _________ effect. a. inotropic b. dromotropic c. chronotropic d. conductivity
answer
b. dromotropic
question
The treatment for sinus tachycardia should focus on: a. decreasing the heart rate. b. correcting the underlying cause. c. administering IV fluid boluses. d. relieving pain and anxiety.
answer
b. correcting the underlying cause.
question
A 41-year-old man complains of chest heaviness and mild shortness of breath that began about 2 hours ago. He is conscious and alert to person, place, time, and event. As you are assessing him, he tells you that he has high blood pressure for which he takes Catapres. His blood pressure is 160/90 mm Hg, heart rate is 148 beats/min and regular, and respirations are 22 breaths/min and somewhat labored. The cardiac monitor displays a narrow complex tachycardia in lead II. Which of the following interventions is NOT indicated for this patient? a. Aspirin b. Adenosine c. IV access d. 12-Lead ECG
answer
b. Adenosine
question
The downslope of the T wave: a. is the point of ventricular repolarization to which a defibrillator is synchronized to deliver electrical energy. b. is the strongest part of ventricular depolarization and is often the origin of dangerous ventricular arrhythmias. c. represents a state of absolute ventricular refractoriness in which another impulse cannot cause depolarization. d. represents a vulnerable period during which a strong impulse could cause depolarization, resulting in a lethal arrhythmia.
answer
d. represents a vulnerable period during which a strong impulse could cause depolarization, resulting in a lethal arrhythmia.
question
You are dispatched to a residence at 4:00 a.m. for an elderly man with shortness of breath. The patient tells you that he was awakened with the feeling that he was smothering. You note traces of dried blood around his mouth. The patient tells you that he has some type of \"breathing problem,\" for which he uses a prescribed inhaler. Based on the information you have obtained thus far, you should be MOST suspicious for: a. cor pulmonale. b. reactive airway disease. c. acute COPD exacerbation. d. left-sided heart failure.
answer
d. left-sided heart failure.
question
A 56-year-old man presents with an acute onset of chest pressure and diaphoresis. He has a history of hypertension and insulin-dependent diabetes. His airway is patent and his breathing is adequate. You should: a. establish vascular access. b. obtain baseline vital signs. c. administer high-flow oxygen. d. acquire a 12-lead ECG tracing.
answer
c. administer high-flow oxygen
question
In contrast to the pain associated with an acute myocardial infarction, pain from a dissecting aortic aneurysm: a. often waxes and wanes. b. gradually becomes severe. c. is maximal from the onset. d. is preceded by other symptoms.
answer
c. is maximal from the onset.
Get an explanation on any task
Get unstuck with the help of our AI assistant in seconds
New